Wellengleichung über D'Alembert lösen

Neue Frage »

kaktus018 Auf diesen Beitrag antworten »
Wellengleichung über D'Alembert lösen
Hallo,
ich habe die Aufgabe die Wellengleichung mit unüblichen Nebenbedingungen zu lösen:


Hier wäre y ja der Ort und x die Zeit. Ich weiß nicht ob da in der Aufgabenstellung was verwechselt wurde, aber das sind ja schon mal eher seltsame Nebenbedingungen.
Jedenfalls lässt sich die Gleichung über die Charakteristiken lösen:

Für die erste Nebenbedingung ergibt sich

Für die zweite Nebenbedingung ergibt sich

Mir fällt jetzt leider nichts ein, wie ich mit diesen beiden Gleichungen auf F_1 und F_2 kommen könnte. Exisitiert da überhaupt eine eindeutige Lösung? Jedenfalls bin ich für jeden Tipp dankbar!
Huggy Auf diesen Beitrag antworten »
RE: Wellengleichung über D'Alembert lösen
Zitat:
Original von kaktus018
Hier wäre y ja der Ort und x die Zeit.

Weshalb? Wenn man und eine physikalische Interpretation geben will, dann ist = Ort und = Zeit doch die natürlichere Wahl. ist dann die Auslenkung am Ort zur Zeit 0 und die Geschwindigkeit am Ort zur Zeit 0.

Zitat:

Ich schreibe die allgemeine Lösung mal leicht anders:



Dann hat man

(1)

(2)

Die letzte Gleichung kann man integrieren. Es ergibt sich

(3)

mit einer Konstanten . Aus (1) und (3) folgt



Das ergibt die Lösung



Das ist auch physikalisch verständlich. Wenn die Anfangsauslenkung überall gleich ist und die Anfangsgeschwindigkeit überall 0 ist, dann bleiben alle Punkte des Körpers dauerhaft in Ruhe.
kaktus018 Auf diesen Beitrag antworten »

Die Wellengleichung lautet eindimensional ja



Aufgrund der Vorzeichen der Ableitungen wäre somit bei meiner Gleichung y als Ort und x als Zeit zu interpretieren. Wie gesagt, ich weiß nicht, ob hier einfach eine Verwechslung in der Aufgabenstellung vorliegt, weil die Forderung y (Ort) >= 0 eher seltsam ist (für die Zeit aber logisch).
Jedenfalls erhalte ich so eben für die erste Nebenbedingung



und für die zweite Nebenbedingung nach Integration



Aus dem Koeffizientenvergleich ergibt sich ja somit direkt C = 1, d.h. die beiden Nebenbedingungen sind abhängig. Deshalb auch meine Frage, ob hier überhaupt eine eindeutige Lösung existiert. Ich könnte ja jetzt höchstens

als Lösung angeben oder verstehe ich hier was falsch?

Vielen Dank schonmal für deine Antwort!
Huggy Auf diesen Beitrag antworten »

Zitat:
Original von kaktus018
Die Wellengleichung lautet eindimensional ja


Multipliziere die Gleichung mit -1 und die Rollen von und sind vertauscht. Die Anfangsbedingungen entsprechen dann dem üblichen Schema. Die Lösung ist dann meiner Erinnerung nach eindeutig, obwohl ich im Moment dazu keinen Satz zitieren kann. Du kannst jedenfalls leicht prüfen, dass die von mir hergeleitete Lösung tatsächlich eine die Anfangsbedingungen erfüllende Lösung ist.
kaktus018 Auf diesen Beitrag antworten »

Zitat:
Multipliziere die Gleichung mit -1 und die Rollen von und sind vertauscht.

Die Gleichung ist natürlich so einfach, dass dies geht. Damit hat sich mein Problem erledigt. Vielen Dank Huggy!
Neue Frage »
Antworten »



Verwandte Themen

Die Beliebtesten »
Die Größten »
Die Neuesten »